1994 AHSME Problems/Problem 24

Revision as of 22:08, 27 June 2014 by TheMaskedMagician (talk | contribs) (Created page with "==Problem== A sample consisting of five observations has an arithmetic mean of <math>10</math> and a median of <math>12</math>. The smallest value that the range (largest observa...")
(diff) ← Older revision | Latest revision (diff) | Newer revision → (diff)

Problem

A sample consisting of five observations has an arithmetic mean of $10$ and a median of $12$. The smallest value that the range (largest observation minus smallest) can assume for such a sample is

$\textbf{(A)}\ 2 \qquad\textbf{(B)}\ 3 \qquad\textbf{(C)}\ 5 \qquad\textbf{(D)}\ 7 \qquad\textbf{(E)}\ 10$

Solution